Home / Logical Reasoning / Statement and Argument :: Statement And Arguments Set - 2 MCQs

Logical Reasoning :: Statement and Argument

  1. Directions : Each of the following question consists of a statement followed by two arguments I and II. You have to decide which of the arguments is a STRONG arguments and which is a WEAK Argument.

    Statement : Should there be only one rate of interest for term deposits of varying durations in banks?

    Arguments :
    I. No. People will refrain from keeping money for longer duration resulting into reduction of liquidity level of banks.
    II. Yes. This will be much simple for the common people and they may be encouraged to keep more money in banks.

  2. A.

     Only argument I is strong

    B.

     Only argument II is strong

    C.

     Either I or II is strong

    D.

     Neither I nor II is strong

    E.

     Both I and II are strong


  3. Directions : Each of the following question consists of a statement followed by two arguments I and II. You have to decide which of the arguments is a STRONG arguments and which is a WEAK Argument.

    Statement : Should all news be controlled by Government in a democracy?

    Arguments :
    I. Yes. Variety of news only confuses people.
    II. No. Controlled news loses credibility.

  4. A.

     Only argument I is strong

    B.

     Only argument II is strong

    C.

     Either I or II is strong

    D.

     Neither I nor II is strong

    E.

     Both I and II are strong


  5. Directions : Each of the following question consists of a statement followed by two arguments I and II. You have to decide which of the arguments is a STRONG arguments and which is a WEAK Argument.

    Statement : Should taxes on colour television be further increased?

    Arguments :
    I. Yes, Colour television is a luxury item and only rich people buy them.
    II. No, Televisions are bought by the poor too.

  6. A.

     Only argument I is strong

    B.

     Only argument II is strong

    C.

     Either I or II is strong

    D.

     Neither I nor II is strong

    E.

     Both I and II are strong


  7. Directions : Each of the following question consists of a statement followed by two arguments I and II. You have to decide which of the arguments is a STRONG arguments and which is a WEAK Argument.

    Statement : Should the educated unemployed youth be paid "unemployment allowance" by the Government?

    Arguments :
    I. Yes. It will provide them some monetary help to either seek employment or to kick-start some 'self-employment' venture.
    II. No. It will dampen their urge to do something to earn their livelihood and thus promote idleness among the unemployed youth.

  8. A.

     Only argument I is strong

    B.

     Only argument II is strong

    C.

     Either I or II is strong

    D.

     Neither I nor II is strong

    E.

     Both I and II are strong


  9. Directions : Each of the following question consists of a statement followed by two arguments I and II. You have to decide which of the arguments is a STRONG arguments and which is a WEAK Argument.

    Statement : Should higher education be restricted to only those who can bear the expenditure?

    Arguments :
    I. Yes. Higher education is very costly; hence it should not be given free.
    II. No. There are a large number of brilliant students who cannot afford to pay and they should be given higher education.

  10. A.

     Only argument I is strong

    B.

     Only argument II is strong

    C.

     Either I or II is strong

    D.

     Neither I nor II is strong

    E.

     Both I and II are strong


  11. Directions : Each of the following question consists of a statement followed by two arguments I and II. You have to decide which of the arguments is a STRONG arguments and which is a WEAK Argument.

    Statement : Should those who receive dowry, despite the law prohibiting it, be punished?

    Arguments :
    I. Yes. Those who violate the law must be punished.
    II. No. Dowry system is firmly rooted in the society since time immemorial.

  12. A.

     Only argument I is strong

    B.

     Only argument II is strong

    C.

     Either I or II is strong

    D.

     Neither I nor II is strong

    E.

     Both I and II are strong


  13. Directions : Each of the following question consists of a statement followed by two arguments I and II. You have to decide which of the arguments is a STRONG arguments and which is a WEAK Argument.

    Statement : Is the Government justified in spending so much on defence?

    Arguments :
    I. Yes. Safety of the country is of prime importance.
    II. No. During peace, this money could be used for the development of the country.

  14. A.

     Only argument I is strong

    B.

     Only argument II is strong

    C.

     Either I or II is strong

    D.

     Neither I nor II is strong

    E.

     Both I and II are strong


  15. Directions : Each of the following question consists of a statement followed by two arguments I and II. You have to decide which of the arguments is a STRONG arguments and which is a WEAK Argument.

    Statement : Should girls learn arts like judo and karate?

    Arguments :
    I. Yes. It will enable them to defend themselves from rogues and ruffians.
    II. No. They will lose their feminine grace.

  16. A.

     Only argument I is strong

    B.

     Only argument II is strong

    C.

     Either I or II is strong

    D.

     Neither I nor II is strong

    E.

     Both I and II are strong


  17. Directions : Each of the following question consists of a statement followed by two arguments I and II. You have to decide which of the arguments is a STRONG arguments and which is a WEAK Argument.

    Statement : Should India develop a national water grid by connecting all the rivers in the country?

    Arguments :
    I. No. This is not just possible as we do not have the technical knowhow.
    II. Yes, this will greatly help the entire country by effectively channelizing the excess water to the areas having shortage.

  18. A.

     Only argument I is strong

    B.

     Only argument II is strong

    C.

     Either I or II is strong

    D.

     Neither I nor II is strong

    E.

     Both I and II are strong


  19. Directions : Each of the following question consists of a statement followed by two arguments I and II. You have to decide which of the arguments is a STRONG arguments and which is a WEAK Argument.

    Statement : Should individuals/institutes having treasures of national significance like Nobel Prizes, hand them over to the Central Government for their safe custody?

    Arguments :
    I. Yes. The individuals or institutions do not have enough resources to protect them.
    II. No. These are the property of the individuals/institutions who win them and should be in their custody.

  20. A.

     Only argument I is strong

    B.

     Only argument II is strong

    C.

     Either I or II is strong

    D.

     Neither I nor II is strong

    E.

     Both I and II are strong